Mathcenter Forum

Mathcenter Forum (https://www.mathcenter.net/forum/index.php)
-   อสมการ (https://www.mathcenter.net/forum/forumdisplay.php?f=18)
-   -   โจทย์หัดแต่งเองครับ ไม่ยาก (https://www.mathcenter.net/forum/showthread.php?t=13443)

LightLucifer 01 เมษายน 2011 22:45

โจทย์หัดแต่งเองครับ ไม่ยาก
 
ให้ $x,y,z>0$ โดยที่ $x^6+y^6+z^6=x^3+y^3+z^3$
จงพิสูจน์ว่า
$$\frac{x^8+y^8+z^8}{3} \geq \frac{x^3+y^3+z^3}{3}\cdot \frac{x^2+y^2+z^2}{3}\cdot \frac{x+y+z}{3}$$

Ne[S]zA 02 เมษายน 2011 01:35

ไม่รู้ผมคิดมากไปหรือเปล่า เฉลยน่าจะสั้นกว่านี้ (นี่ก็ยังไม่มั่นใจว่าถูกหรือเปล่า:p)
โดย Cauchy's inequality จะได้ว่า $x^3+y^3+z^3\leqslant \sqrt{x^6+y^6+z^6} \sqrt{1+1+1}$
นั่นคือ $x^6+y^6+z^6 \geqslant \dfrac{(x^3+y^3+z^3)^2}{3}$
จาก $x^6+y^6+z^6=x^3+y^3+z^3$ เพราะฉะนั้น $1 \geqslant \dfrac{x^3+y^3+z^3}{3}$
จะได้ว่า $\dfrac{x^2+y^2+z^2}{3} \geqslant \dfrac{x^3+y^3+z^3}{3} \cdot \dfrac{x^2+y^2+z^2}{3}\_\_\_\_\_\_\_(*)$
โดย Cauchy's inequality จะได้ว่า
$\dfrac{(x+y+z)^2}{3}\leqslant x^2+y^2+z^2\_\_\_\_(I)$
$\dfrac{(x^2+y^2+z^2)^2}{3}\leqslant x^4+y^4+z^4\_\_\_\_(II)$
$\dfrac{(x^4+y^4+z^4)^2}{3}\leqslant x^8+y^8+z^8\_\_\_\_(III)$
จาก (II) และ (III) จะได้ว่า $x^8+y^8+z^8\geqslant \dfrac{(x^2+y^2+z^2)^4}{3^3}$ จะได้ว่า $\dfrac{x^8+y^8+z^8}{3} \geqslant (\dfrac{x^2+y^2+z^2}{3})^4=\dfrac{x^2+y^2+z^2}{3} \cdot (\dfrac{x^2+y^2+z^2}{3})^3 $
จาก (*) จะได้ว่า $\dfrac{x^8+y^8+z^8}{3} \geqslant \dfrac{x^3+y^3+z^3}{3} \cdot \dfrac{x^2+y^2+z^2}{3} \cdot (\dfrac{x^2+y^2+z^2}{3})^3 $
จาก (I) จะได้ว่า $\dfrac{x^8+y^8+z^8}{3} \geqslant \dfrac{x^3+y^3+z^3}{3} \cdot \dfrac{x^2+y^2+z^2}{3} \cdot (\dfrac{x+y+z}{3})^6 $
ต่อไปจะแสดงว่า $x+y+z \geqslant 3$ สำหรับ $x,y,z>0$
เนื่องจาก $f(x,y,z)=x+y+z$ เป็น Homogeneous จึงสมมติ $xyz=1$
โดย AM-GM inequality จะได้ว่า $x+y+z\geqslant 3 (xyz)^{1/3}=3$
จึงได้ว่า $(x+y+z)^5\geqslant 3^5$ นั่นคือ $(\dfrac{x+y+z}{3})^5\geqslant 1$
เพราะฉะนั้น $(\dfrac{x+y+z}{3})^6\geqslant \dfrac{x+y+z}{3}$
ดังนั้น $\dfrac{x^8+y^8+z^8}{3} \geqslant \dfrac{x^3+y^3+z^3}{3} \cdot \dfrac{x^2+y^2+z^2}{3} \cdot \dfrac{x+y+z}{3} $

จูกัดเหลียง 02 เมษายน 2011 08:20

$x^3+y^3+z^3\geqslant \sqrt{x^6+y^6+z^6} \sqrt{1+1+1}$

ผมว่า มันกลับด้านป่ะครับ หรือผมเข้าใจอะไรผิด

จูกัดเหลียง 02 เมษายน 2011 08:55

ส่วนทีผมคิดครับ ยังไงก็ช่วยเช็คหน่อยครับ

LightLucifer 02 เมษายน 2011 10:14

#2 ผมไม่ค่อยแน่ใจนะครับเพราะวิธีนี้ไม่ค่อยได้ใช้ ถ้าเข้าใจผิดก็ชี้แนะด้วย
แต่ว่าถ้าจะกำหนด $xyz=1$ อสมการเวลาคูณค่าคงตัวเข้าไปต้องกลับเป็นรูปเดิมไม่ใช่เหรอครับ แต่อันนี้ดีกรีฝั่งขวามัน 11 แต่ ฝั่งซ้ายมัน 8 อ่ะครับ
#4 ช่วยเรียบเรียงใหม่หน่อยได้ไหมครับ ผมอ่านไม่ค่อยเข้าใจ

Ne[S]zA 02 เมษายน 2011 11:01

อ้างอิง:

ข้อความเดิมเขียนโดยคุณ จูกัดเหลียง (ข้อความที่ 114301)
$x^3+y^3+z^3\geqslant \sqrt{x^6+y^6+z^6} \sqrt{1+1+1}$

ผมว่า มันกลับด้านป่ะครับ หรือผมเข้าใจอะไรผิด

พิมพ์ผิดครับ :D
ส่วนที่สมมติ xyz=1 เดี๋ยวขอไปดูใหม่

nooonuii 02 เมษายน 2011 12:27

$\dfrac{x^8+y^8+z^8}{3}\geq \Big(\dfrac{x^6+y^6+z^6}{3}\Big)^{8/6}\geq \Big(\dfrac{x^3+y^3+z^3}{3}\Big)^2\geq \dfrac{x^3+y^3+z^3}{3}\cdot \dfrac{x^2+y^2+z^2}{3}\cdot \dfrac{x+y+z}{3}$

Power mean - เงื่อนไขโจทย์ - chebychev

เติมให้ จะได้ต่อเนื่อง

$a,b,c>0$

$\Big(a^7+b^7+c^7\Big)\Big(\dfrac{1}{a^5}+\dfrac{1}{b^5}+\dfrac{1}{c^5}\Big)\geq \Big(a^5+b^5+c^5\Big)\Big(\dfrac{1}{a^3}+\dfrac{1}{b^3}+\dfrac{1}{c^3}\Big)$

LightLucifer 02 เมษายน 2011 15:44

อ้างอิง:

ข้อความเดิมเขียนโดยคุณ nooonuii (ข้อความที่ 114317)
$\dfrac{x^8+y^8+z^8}{3}\geq \Big(\dfrac{x^6+y^6+z^6}{3}\Big)^{8/6}\geq \Big(\dfrac{x^3+y^3+z^3}{3}\Big)^2\geq \dfrac{x^3+y^3+z^3}{3}\cdot \dfrac{x^2+y^2+z^2}{3}\cdot \dfrac{x+y+z}{3}$

Power mean - เงื่อนไขโจทย์ - chebychev

เติมให้ จะได้ต่อเนื่อง

$a,b,c>0$

$\Big(a^7+b^7+c^7\Big)\Big(\dfrac{1}{a^5}+\dfrac{1}{b^5}+\dfrac{1}{c^5}\Big)\geq \Big(a^5+b^5+c^5\Big)\Big(\dfrac{1}{a^3}+\dfrac{1}{b^3}+\dfrac{1}{c^3}\Big)$

เจ๋งกว่าเฉลยอีกครับ 555+
ส่วนข้อของคุณ nooonuii ผมได้แบบนี้อ่ะครับ
ข้อนี้เป็นโจทย์แรกที่ผมคิดเองครับ เคยโพสไปแล้วในห้องมาราธอน แต่ยังไม่มีใครทำ ลองทำดูนะครับ

ให้ $x,y,z\in \mathbb{R} $ โดยที่ $\sum_{cyc}(x^2+y^2)(y^2+z^2) =12$
จงพิสูจน์ว่า
$$\frac{1}{3x^2+2x+1}+\frac{1}{3y^2+2y+1}+\frac{1}{3z^2+2z+1}+\frac{27}{4(x^2+y^2+z^2)+42} \geq 1$$

LightLucifer 03 เมษายน 2011 11:49

ขอเปลี่ยนโจทย์ดีกว่าครับ
กลัวจะร้างเหมือน มาราธอน :sweat::sweat:
ข้อนี้เรียบๆครับ ไม่น่ากลัว เพิ่งคิดได้ร้อนๆเลย (ตอนนี้รู้แล้วว่าการสร้างโจทย์เรียบๆสวยๆ ยากกว่าการสร้างโจทย์แบบข้อข้างบน:sweat: )


ให้ $a,b,c>0$ โดยที่ $3 \ge \frac{1}{a}+\frac{1}{b}+\frac{1}{c}$
จงพิสูจน์ว่า $$a+b+c \ge \sqrt{a}+\sqrt{b}+\sqrt{c}$$

Ne[S]zA 03 เมษายน 2011 15:18

โดย Cauchy's Inequality จะได้ว่า $a+b+c \ge \sqrt{ab}+\sqrt{bc}+\sqrt{ca}$
โดย Muirhead's Inequality จะได้ว่า $\sum_{sym} a^{\frac{1}{2}}b^{\frac{1}{2}}c^0\geqslant \sum_{sym} a^{\frac{1}{2}}b^0c^0$ นั่นคือ $\sqrt{ab}+\sqrt{bc}+\sqrt{ca} \geqslant
\sqrt{a}+\sqrt{b}+\sqrt{c}$
เพราะฉะนั้น $a+b+c\geqslant \sqrt{a}+\sqrt{b}+\sqrt{c}$
เนื่องจาก $a\geqslant \sqrt{a},b \geqslant \sqrt{b}$ และ $c\geqslant \sqrt{c}$ เมื่อ $a,b,c\geqslant1$ นั่นคือ $3\geqslant \dfrac{1}{a}+\dfrac{1}{b}+\dfrac{1}{c}$
เพราะฉะนั้น $a+b+c\geqslant \sqrt{a}+\sqrt{b}+\sqrt{c}$ เป็นจริงเมื่อ $3\geqslant \dfrac{1}{a}+\dfrac{1}{b}+\dfrac{1}{c}$

LightLucifer 03 เมษายน 2011 15:34

Muirhead's Inequality สองข้างต้องดีกรีเท่ากันครับ

Ne[S]zA 03 เมษายน 2011 16:18

จริงด้วย TT

ทำงี้ได้มั้ยครับ
จาก $3\geqslant \dfrac{1}{a}+\dfrac{1}{b}+\dfrac{1}{c}$
ได้ว่า $3(a+b+c)\geqslant (a+b+c)(\dfrac{1}{a}+\dfrac{1}{b}+\dfrac{1}{c})=1+1+1+\dfrac{b+c}{a}+\dfrac{c+a}{b}+\dfrac{a+b}{c}\geqslant 9$ จะได้ว่า $a+b+c\geqslant 3\_\_\_\_\_(I)$
โดย Cauchy's Inequality ได้ว่า
$(a+b+c)\geqslant \dfrac{(\sqrt{a}+\sqrt{b}+\sqrt{c})^2}{3}$
จาก (I) ได้ว่า $9\geqslant (\sqrt{a}+\sqrt{b}+\sqrt{c})^2$ ได้ว่า $3 \geqslant \sqrt{a}+\sqrt{b}+\sqrt{c}\_\_\_\_\_(II)$
จาก (I) และ (II) ได้ว่า $a+b+c\geqslant \sqrt{a}+\sqrt{b}+\sqrt{c}$
แก้แล้วครับ

LightLucifer 03 เมษายน 2011 16:54

ตรง Cuachy Schwarz inequality มาได้ยังไงเหรอครับ
แล้วก็ (II) แทน $a=b=c=1$ แล้วไม่จริงครับ

Ne[S]zA 03 เมษายน 2011 17:01

มึนอีกแล้ว ผิดหมดเลยครับ T T

แก้แล้วครับ

LightLucifer 03 เมษายน 2011 17:25

ทำไม $9 \ge (\sqrt{a}+\sqrt{b}+\sqrt{c})^2$ อ่ะครับ


เวลาที่แสดงทั้งหมด เป็นเวลาที่ประเทศไทย (GMT +7) ขณะนี้เป็นเวลา 00:53

Powered by vBulletin® Copyright ©2000 - 2024, Jelsoft Enterprises Ltd.
Modified by Jetsada Karnpracha